Sei sulla pagina 1di 21

Abstract Algebra

Chapter 13 - Field Theory


David S. Dummit & Richard M. Foote

Solutions by positrón0802
positron0802@mail.com

March 31, 2018

Contents
13 Field Theory 1
13.1 Basic Theory and Field Extensions . . . . . . . . . . . . . . . . . . . . . . . . . . . 1
13.2 Algebraic Extensions . . . . . . . . . . . . . . . . . . . . . . . . . . . . . . . . . . 4
13.3 Classical Straightedge and Compass Constructions . . . . . . . . . . . . . . . . . . 10
13.4 Splitting Fields and Algebraic Closures . . . . . . . . . . . . . . . . . . . . . . . . 12
13.5 Separable and Inseparable Extension . . . . . . . . . . . . . . . . . . . . . . . . . . 13
13.6 Cyclotomic Polynomials and Extensions . . . . . . . . . . . . . . . . . . . . . . . . 17

13 Field Theory
13.1 Basic Theory and Field Extensions
Exercise 13.1.1.
p(x) = x 3 + 9x + 6 is irreducible in Z[x] by Eisenstein Criterion with p = 3. By Gauss Lemma, then
it is irreducible in Q[x]. To find (1 + θ) −1 , we apply the Euclidean algorithm (long division) to p(x)
and 1 + x. We find
x 3 + 9x + 6 = (1 + x)(x 2 − x + 10) − 4.
Evaluating at θ, we find (1 + θ)(θ 2 − θ + 10) = 4. Therefore
θ 2 − θ + 10
(1 + θ) −1 = .
4

Exercise 13.1.2.
Let f (x) = x 3 − 2x − 2. f is irreducible over Z by Eisenstein Criterion with p = 2, hence over Q by
Gauss Lemma. Now, if θ is a root of f , then θ 3 = 2θ + 2. Hence

(1 + θ)(1 + θ + θ 2 ) = 1 + 2θ + 2θ 2 + θ 3 = 3 + 4θ + 2θ 2 .
1+θ
For computing , first we compute (1 + θ + θ 2 ) −1 . Applying the Euclidean algorithm, we
1 + θ + θ2
obtain
x 3 − 2x − 2 = (x 2 + x + 1)(x − 1) − 2x − 1,
and
x2 x 7 9
x 3 − 2x − 2 = (2x + 1)( − − )− .
2 4 8 8

1
13.1 Basic Theory and Field Extensions

Evaluating at θ, from this equalities we obtain

8 θ2 θ 7
(θ 2 + θ + 1)(θ − 1) = 2θ + 1 and (2θ + 1) −1 = ( − − ).
9 2 4 8
Combining these two equations we obtain

8 2 θ2 θ 7
(θ + θ + 1)(θ − 1)( − − ) = 1.
9 2 4 8
So,
8 θ2 θ 7 2θ 2 θ 5
(θ 2 + θ + 1) −1 = (θ − 1)( − − ) = − + + ,
9 2 4 8 3 3 3
where we used θ 3 = 2θ + 2 again. Therefore,

1+θ 2θ 2 θ 5 θ 2 2θ 1
= (1 + θ)(− + + ) = − + + .
1 + θ + θ2 3 3 3 3 3 3

Exercise 13.1.3.
Since 03 + 0 + 1 = 1 and 11 + 1 + 1 = 1 in F2 , then x 3 + x + 1 is irreducible over F2 . Since θ is root
of x 3 + x + 1, then θ 3 = −θ − 1 = θ + 1. Hence, the powers of θ in F2 (θ) are

θ, θ 2, θ 3 = θ + 1, θ 4 = θ 2 + θ, θ 5 = θ 2 + θ + 1, θ 6 = θ 2 + 1, and θ 7 = 1.

Exercise 13.1.4.
Denote this map by ϕ. Then
√ √ √ √ √ √
ϕ(a + b 2 + c + d 2) = a + c − b 2 − d 2 = ϕ(a + b 2) + ϕ(c + d 2),

and √ √ √
ϕ((a + b 2) · (c + d 2)) = ϕ(ac + 2bd + (ad + bc) 2)

= ac + 2bd − (ad + bc) 2
√ √
= (a − b 2)(c − d 2)
√ √
= ϕ(a + b 2)ϕ(c + d 2),
√ √ √ √
hence ϕ is an√homomorphism. Moreover, if ϕ(a + b 2) = ϕ(c + d 2), then a √ − b 2 =√c − d 2,
hence (since
√ √ a = b and c = d, so ϕ is injective. Also, given a + b√ 2 ∈ Q( 2), then
2 < Q)
ϕ(a − b 2) = a + b 2, so ϕ is surjective. Therefore, ϕ is an isomorphism of Q( 2) with itself.
Exercise 13.1.5.
Let α = p/q be a root of a monic polynomial p(x) = x n + · · · + a1 x + a0 over Z, with gcd(p, q) = 1.
Then
p p p
( ) n + an−1 ( ) n−1 + · · · + a1 + a0 = 0.
q q q
Multiplying this equation by q n one obtains

pn + an−1 pn−1 q + · · · + a1 pq n−1 + a0 q n = 0


⇒ q(an−1 pn−1 + · · · + a1 pq n−2 + a0 q n−1 ) = −pn .

Thus, every prime that divides q divides pn as well, so divides p. Since gcd(p, q) = 1, there is no
prime dividing q, hence q = ±1. The result follows.

2
13.1 Basic Theory and Field Extensions

Exercise 13.1.6.
This is straightforward. If

an α n + an−1 α n−1 + · · · + a1 α + a0 = 0,

then
(an α) n + an−1 (an α) n−1 + an an−2 (an α) n−2 + · · · + ann−2 a1 (an α) + ann−1 a0
= ann α n + ann−1 an−1 α n−1 + ann−1 an−2 α n−2 + · · · + ann−1 a1 α + ann−1 a0
= ann−1 (an α n + an−1 α n−1 + an−2 α n−2 + · · · + a1 α + a0 ) = 0.

Exercise 13.1.7.
If − nx + 2 is reducible it must have a linear factor, hence a root. By the Rational Root Theorem,
x3
if α is a root of x 3 − nx + 2, then α must divide its constant term, so the possibilities are α = ±1, ±2.
If α = −1 or 2, then n = 3; if α = −1, then n = 1; and if α = 2, then n = 5. Therefore, x 3 − nx + 2 is
irreducible for n , −1, 3, 5.
Exercise 13.1.8.
We subdivide this exercise in cases and subcases.
If x 5 − ax − 1 is reducible then it has a root (linear factor) or is a product of two irreducible
polynomials of degrees 2 and 3.
Case 1. If x 5 − ax − 1 has a root, then, by the Rational Root Theorem, it must be α = ±1. If
α = 1 is a root, then a = 0. If α = −1 is a root, then a = 2.
Case 2. Now, suppose that there exists f (x) and g(x) irreducible monic polynomials over Z of
degrees 2 and 3 respectively, such that x 5 − ax − 1 = f (x)g(x). Write f (x) = x 2 + bx + c and
g(x) = x 3 + r x 2 + sx + t, where b, c, r, s, t ∈ Z. Then

x 5 − ax − 1 = (x 2 + bx + c)(x 3 + r x 2 + sx + t)
= x 5 + (b + r)x 4 + (br + c + s)x 3 + (bs + cr + t)x 2 + (bt + cs) + tc.
Equating coefficients leads to
b+r = 0
br + c + s = 0
bs + cr + t = 0
bt + cs = −a
ct = −1.
From ct = −1 we deduce (c, t) = (−1, 1) of (c, t) = (1, −1), which give us two cases.
Case 2.1. First suppose (c, t) = (−1, 1). Then the system of equations reduces to

b+r = 0
br − 1 + s = 0
bs − r + 1 = 0
b − s = −a.

Now, put b = −r into second and third equations to obtain −r 2 − 1 + s = 0 and −r s − r + 1 = 0, that
is, r 2 + 1 − s = 0 and r s + r − 1 = 0. Adding these last two equations we obtain r 2 + r s + r − s = 0.
Thus r 2 + r s + r + s = 2s, so (r + 1)(r + s) = 2s. Now, from r 2 + 1 − s = 0 we have r 2 = s − 1, so
then r 2 + r s + r − s = 0 becomes r s + r = 1, that is, r (s + 1) = 1. Hence, r = 1 and s = 0, or r = −1
and s = −2. If r = 1 and s = 0, then (r + 1)(r + s) = 2s leads to 2 = 0, a contradiction. If r = −1
and s = −2, it leads to 0 = −4, another contradiction.

3
13.2 Algebraic Extensions

Therefore, (c, t) = (−1, 1) is impossible. We now pass to the case (c, t) = (1, −1).
Case 2.2. Suppose that (c, t) = (1, −1). The system of equations reduces to
b+r = 0
br + 1 + s = 0
bs + r − 1 = 0
−b + s = −a.
Adding the second and third equation we obtain b(r + s) + r + s = 0, so that (b + 1)(r + s) = 0. Then
b = −1 or r = −s, so one more time we have two cases. If r = −s, then br + 1 + s = 0 becomes
br + 1 − r = 0. Hence, b = −r and br + 1 − r = 0 gives r 2 + r − 1 = 0. By the Rational Root Theorem,
this equation has no roots on Z. Since r ∈ Z, we have a contradiction. Now suppose b = −1. From
b = −r we obtain r = 1, so, from br + 1 + s = 0 we obtain s = 0. Finally, from −b + s = −a we
obtain a = −1. Therefore, the solution (b, c, r, s, t) = (−1, 1, 1, 0, −1) is consistent and we obtain the
factorization

x 5 − ax − 1 = (x 2 + bx + c)(x 3 + r x 2 + sx + t) = (x 2 − x + 1)(x 3 + x 2 − 1).

13.2 Algebraic Extensions


Exercise 13.2.1.
Since the characteristic of F is p, its prime subfield is (isomorphic to) F p = Z/pZ. We can consider
F as a vector space over F p . Since F is finite, then [F : F p ] = n for some n ∈ Z+ . Therefore

|F| = |F p | [F:F p ] = pn .

Exercise 13.2.2.
Note that g and h are irreducible over F2 and F3 . Now, is θ is a root of g, then F2 (θ)  F2 /(g(x))
has 4 elements and F3 (θ)  F3 /(g(x)) has 9 elements. Furthermore, is θ 2 is a root of h, then
F2 (θ 2 )  F2 /(h(x)) has 8 elements and F3 (θ 2 )  F3 /(h(x)) has 27 elements.
The multiplication table for F2 /(g(x)) is

· 0 1 x x+1
0 0 0 0 0
1 0 1 x x+1
x 0 x x+1 x
x+1 0 x+1 x x

The multiplication table for F3 /(g(x)) is

· 0 1 2 x x+1 x+2 2x 2x + 1 2x + 2
0 0 0 0 0 0 0 0 0 0
1 0 1 2 x x+1 x+2 2x 2x + 1 2x + 2
2 0 2 1 2x 2x + 2 2x + 1 x x+2 x+1
x 0 x 2x 2x + 1 1 x+1 x+2 2x + 2 2
x+1 0 x+1 2x + 2 1 x+2 2x 2 x 2x + 1
x+2 0 x+2 2x + 1 x+1 2x 2 2x + 2 1 x
2x 0 2x x x+2 2 2x + 2 2x + 1 x+1 1
2x + 1 0 2x + 1 x+2 2x + 2 x 1 x+1 2 2x
2x + 2 0 2x + 2 x+1 2 2x + 1 x 1 2x x+2

4
13.2 Algebraic Extensions

In both cases, x is a generator of the cyclic group of nonzero elements.


Exercise 13.2.3.
Since 1 + i < Q, its minimal polynomial is of degree at least 2. We try conjugation, and obtain

(x − (1 + i))(x − (1 − i)) = x 2 − 2x + 2,

which is irreducible by Eisenstein with p = 2. Therefore, the minimal polynomial is x 2 − 2x + 2.


Exercise 13.2.4.
√ √ √ √ √ √
First, note that (2+ 3) 2 = 4+4 3+3 = 7+4 3. Let θ = 2+ 3. Then θ 2 −4θ = 7+4 3−8−4 3 = −1,
hence θ is a root of x 2 − 4x + 1. Moreover, x√2 − 4x + 1 is irreducible
√ over Q (because θ < Q), so
x − 4x + 1 is the minimal
2
√3 polynomial of 2 + 3. Therefore, 2 + 3 has degree 2 over Q.
Now, let α = 2 and β = 1 + α + α 2 . Then β ∈ Q(α), so Q ⊂ Q( β) ⊂ Q(α). We have
[Q(α) : Q] = [Q(α) : Q( β)][Q( β) : Q]. Note that [Q(α) : Q] = 3 since α has minimal polynomial
x 3 − 2 over Q, so [Q( β) : Q] = 1 or 3. For a contradiction, suppose [Q( β) : Q] = 1, that is, Q( β) = Q
so β ∈ Q. Then

β 2 = (1 + α + α 2 ) 2 = 1 + 2α + 3α 2 + 2α 3 + α 4 = 5 + 4α + 3α 2,

where we used α 3 = 2. So

β 2 − 3 β = 5 + 2α + 3α 2 − 3(1 + α + α 2 ) = 2 − α,

hence α = − β 2 + 3 β + 2 ∈ Q( β) = Q, a contradiction. Therefore, [Q( β) : Q] = 3.


Exercise 13.2.5.
Since the polynomials have degree 3, if they were reducible they must have a linear factor, hence
a root in F.√ Note √3 element of F is of the form a + bi, where a, b ∈ Q. The roots of
√3 that every
x − 2 are 2, ζ 2 and ζ 2, where ζ is the primitive 3’rd root of unity, i.e., ζ = exp(2πi/3) =
3 3 2
√ √
cos(2π/3) + i sin(2π/3) = − 2 + 2 . Since 3 < Q, none of this elements is in F, hence x 3 − 2 is
1 3
√3 √3 √3
irreducible over F. Similarly, the roots of x 3 − 3 are 3, ζ 3 and ζ 2 3, and by the same argument
non of this elements is in F. Hence x 3 − 3 is irreducible over F.
Exercise 13.2.6.
We have to prove that F (α1, . . . , α n ) is the smallest field containing F (α1 ), . . . , F (α n ). Clearly
F (αi ) ⊂ F (α1, . . . , α n ) for all 1 ≤ i ≤ n. Now let K be a field such that F (αi ) ⊂ K for all i. If θ
is an element of F (α1, . . . , α n ), then θ is of the form θ = a1 α1 + · · · + an α n , where a1, . . . , an ∈ F.
Every ai αi is in K, hence θ ∈ K. Thus F (α1, . . . , α n ) ⊂ K. Therefore, F (α1, . . . , α n ) contains all
F (αi ) and is contained in every field containing all F (αi ), hence F (α1, . . . , α n ) is the composite of
the fields F (α1 ), F (α2 ), . . . , F (α n ).
Exercise 13.2.7.
√ √ √ √ √ √ √ √
Since 2 + √3 is in Q(√ 2, 3), clearly
√ √Q( 2 + 3) ⊂
√ Q(√ 2, 3). For the other direction we have
to prove that 2 and 2 are in Q( 2 + 3). Let θ = 2 + 3. Then
√ √ √ √
θ 2 = 5 + 2 6, θ 3 = 11 2 + 9 3 and θ 4 = 37 + 15 6.

So
√ 1 √ 1 √
2 = (θ 3 − 9θ), 3 = (11θ − θ 3 ) and θ 4 = 49 + 20 6.
2 2
√ √ √ √ √ √
Therefore
√ √ 3√ ∈ Q(θ), so Q( 2, 3) ⊂ Q( 2 + 3). The equality follows. Hence,
√ 2 ∈ Q(θ) and
[Q( 2 + 3) : Q] = [Q( 2, 3) : Q] = 4.

5
13.2 Algebraic Extensions

We also have
√ √
θ 4 − 10θ 2 = (49 + 20 6) − 10(5 + 2 6) = −1, so θ 4 − 10θ 2 + 1 = 0.
√ √ √ √
Since [Q( 2 + 3) : Q] = 4, then x 4 − 10x 2 + 1 is irreducible over Q, and is satisfied by 2 + 3.
Exercise 13.2.8.
√ √
The elements of F ( D1, D2 ) can be written in the form

a + b D1 + c D2 + d D1 D2, where a, b, c, d ∈ F.
p p p

We have
[F ( D1, D2 ) : F] = [F ( D1, D2 ) : F ( D1 )][F ( D1 ) : F].
p p p p p p
√ √ √ √ √
Since [F ( D1 ) : √F] =√2, then [F√( D1, D2 ) : F] can be 2 or 4. Now, [F ( D1, D2 ) : F] = 2
√ only if [F ( D√1, D2 ) :√F ( D1 )] = 1, and that occurs exactly when x − D2 is reducible in
if and 2

F ( D1 ) (i.e., when D2 ∈ F ( D1 )), that is, if there exists a, b ∈ F such that

(a + b D1 ) 2 = D2, so that a2 + 2ab D1 + b2 D12 = D2 .


p p


Note that ab = 0 as ab , 0 implies D1 ∈ F, contrary to the hypothesis. Then a = 0 or b = 0. If
b = 0, then D2 is a square in F, contrary to the hypothesis. If a = 0,√then b2 D1 = D2 , and thus
D1 D2 = ( Db2 ) 2 , so D1 D2 is a square in F. So, x 2 − D2 is reducible in F ( D1 ) if and only if D1 D2 is
a square in F. The result follows.
Exercise
q 13.2.9.
√ √ √ √ √
Suppose a + b = m + n for some m, n ∈ F, then a + b = m + n + 2 mn. Since b is not a
√ √
q √ √ √
square in F, this means b = 2 mn. We also have a + b − n = m, so

√ √
q
√ √
b = 2 n( a + b − n).

Hence,
√ q √
b = 2 n(a + b) − 2n
√ √
⇒ ( b + 2n) 2 = 4n(a + b)
√ √
⇒ b + 4n b + 4n2 = 4n(a + b)
⇒ b + 4n2 − 4na = 0

4a ± 16a2 − 16b
⇒ n=
8
p 2n
⇒ a2 − b = ± .
a

Therefore, since a and n are in F, a2 − b is in F. √
Now suppose that a2 − b is a square in F, so that a2 − b ∈ F. We prove that there exists m, n ∈ F
q √ √ √
such that a + b = m + n. Let
√ √
a + a2 − b a − a2 − b
m= and n= .
2 2

6
13.2 Algebraic Extensions

q √ √ √
Note that m and n are in F as char(F) , 2. We claim a + b = m + n. Indeed, we have

√ √ √
q √ q √
(a + b) + 2 a2 − b + (a − b) a+ b+ a− b
m= =( ) 2,
4 2
and
√ √ √
q √ q √
(a + b) − 2 a2 − b + (a − b) a+ b− a− b
n= =( )2 .
4 2
Thus q √ q √ q √ q √
√ a+ b+ a− b √ a+ b− a− b
m= and n= .
2 2
Therefore,
q √ q √ q √ q √
a+ b+ a− b a+ b− a− b √
q
√ √
m+ n= + = a + b,
2 2
as claimed. q √
Now, we this to determine when the field Q( a + b), a, b ∈ Q, is biquadratic over Q. If a2 − b
q √ √ √ √ √
is a square in Q and b is not, we have Q( a + b) = Q( m + n) = Q( m, n), so by last exercise
q √
Q( a + b) is biquadratic over Q when a2 − b is a square in Q, and neither b, m, n or mn are squares
in Q. Since √ √
a + a2 − b a − a2 − b b
mn = = ,
2 2 4
q √
then mn is never a square when b isn’t. Thus, Q( a + b) is biquadratic over Q exactly when a2 − b
is a square in Q and neither b, m nor n is a square in Q.
Exercise
q 13.2.10.
√ q √
Note that 3 + 2 2 = 3 + 8. Recalling last exercise with a = 3 and b = 8, we have a2 − b =
9 − 8 = 1 is a square in Q and b q
= 8 is not. Hence, we find (m = 2 and n = 1 from last
q exercise)
q √ √ √ √ √
3 + 8 = 2 + 1. Therefore, Q( 3 + 2 2) = Q( 2) and the degree of the extension Q( 3 + 2 2)
over Q is 2.
Exercise 13.2.11.
(a) First, note that the conjugation map a+bi → a−bi is an isomorphism of C, so it takes squares √ roots
to square roots, and maps numbers of the first quadrant to the fourth (and reciprocally). Since 3 + 4i
√ of 3 + 4i in the√first quadrant,
is the square root √ its conjugate is the square of root of 3 − 4i in the fourth
quadrant, so is 3 − 4i. Hence 3 + 4i and 3 − 4i are conjugates each other. Now, we use Exercise

q √
9 again. Note that 3 + 4i = 3 + −16. With a = 3 and b = −16, we have a2 − b = 25 is a square
√ √
in Q and b = −16 is not. √ Hence, we find m = 1 and
√ n = −4 √ and thus 3 + 4i
√ = 1 + −4 = 1 + 2i.

Furthermore, we find 3 − 4i = 1 − 2i. Therefore, 3 + 4i + 3 − 4i = 4, i.e., 3 + 4i + 3 − 4i ∈ Q.
q √ q √
(b) Let θ = 1 + −3 + 1 − −3. Then

√ √ √ √ √
q q
θ = ( 1 + −3 + 1 − −3) 2 = (1 + −3) + (2 1 + 3) + (1 − −3) = 6.
2

Since x 2 − 6 is irreducible over Q (Eisenstein p = 2), then θ has degree 2 over Q.

7
13.2 Algebraic Extensions

Exercise 13.2.12.
Let E be a subfield of K containing F. Then

[K : F] = [K : E][E : F] = p.

Since p is prime, either [K : E] = 1 or [E : F] = 1. The result follows.


Exercise 13.2.13.
Note that, for all 1 ≤ k ≤ n,√we have [Q(α1, . . . , α√
k ) : Q(α 1, . . . , α√
k−1 )] = 1 or 2. Then [F : Q] = 2
m
3 3 3
for some m ∈ N. Suppose 2 ∈ F. Then √3 Q ⊂ Q( 2) ⊂ F, so [Q( 2) : Q] divides [F : Q], that is, 3
divides 2m , a contradiction. Hence, 2 < F.
Exercise 13.2.14.
Since α 2 ∈ F (α), clearly F (α 2 ) ⊂ F (α). Thus we have to prove α ∈ F (α 2 ). For this purpose,
consider the polynomial p(x) = x 2 − α 2 , so that p(α) = 0. Note that α ∈ F (α 2 ) if and only
if p(x) is reducible in F (α 2 ). For a contradiction, suppose p(x) is irreducible in F (α 2 ), so that
[F (α) : F (α 2 )] = 2. Thus

[F (α) : F] = [F (α) : F (α 2 )][F (α 2 ) : F] = 2[F (α 2 ) : F],

so [F (α) : F] is even, a contradiction. Therefore, p(x) is reducible in F (α 2 ) and α ∈ F (α 2 ).


Exercise 13.2.15.
We follow the hint. Suppose there exists a counterexample. Let α be of minimal degree such that
F (α) is not formally real and α having minimal polynomial f of odd degree, say deg f = 2k + 1
for some k ∈ N. Since F (α) is not formally real, then −1 can be express as a sum of squares in
F (α)  F[x]/(( f (x))). Then, the exists polynomials p1 (x), . . . , pm (x), g(x) such that

−1 + f (x)g(x) = (p1 (x)) 2 + · · · + (pm (x)) 2 .

As every element in F[x]/(( f (x)) can be written as a polynomial in α with degree less than deg f ,
we have deg pi < 2k + 1 for all i. Thus, the degree in the right hand of the equation is less than
4k + 1, so deg g < 2k + 1 as well. We prove that the degree of g is odd by proving that the degree of
(p1 (x)) 2 +· · ·+(pm (x)) 2 is even, because then the equation −1+ f (x)g(x) = (p1 (x)) 2 +· · ·+(pm (x)) 2
implies the result. Let d be the maximal degree over all pi , we prove that x 2d is the leading term of
(p1 (x)) 2 + · · · + (pm (x)) 2 . Note that x 2d is a sum of squares (of the leading coefficients of the pi ’s of
maximal degree). Now, since F is formally real, 0 can’t be expressed as a sum of squares in F. Indeed,
if li=1 ai2 = 0, then l−1 i=1 (ai /al ) = −1. Therefore x
2 2d , 0, so the degree of (p (x)) 2 +· · ·+ (p (x)) 2
P P
1 m
is 2d, as claimed. Hence, the degree of g must be odd by the assertion above. Then g must contain
an irreducible factor of odd degree, say h(x). Since deg g < deg f , we have deg h < deg f as well.
Let β be a root of h(x), hence a root of g(x). Then
f (x)g(x)
−1 + h(x) = (p1 (x)) 2 + · · · + (pm (x)) 2,
h(x)
so −1 is a square in F[x]/((h(x))  F ( β), which means F ( β) is not formally real. Therefore, β is
a root of an odd degree polynomial h such that F ( β) is not formally real. Since deg h < deg f , this
contradicts the minimality of α. The result follows.
Exercise 13.2.16.
Let r ∈ R be nonzero. Since r is algebraic over F, there exist an irreducible polynomial p(x) =
a0 + a1 x + · · · + x n ∈ F[x] such that p(r) = 0. Note that a0 , 0 since p is irreducible. Then
r −1 = −a0−1 (r n−1 + · · · + a1 ). Since ai ∈ F ⊂ R and r ∈ R, we have r −1 ∈ R.

8
13.2 Algebraic Extensions

Exercise 13.2.17.
Let p(x) be an irreducible factor of f (g(x)) of degree m. Let α be a root of p(x). Since p is
irreducible, then [F (α) : F] = deg p(x) = m. Now, since p(x) divides f (g(x)), we have f (g(α)) = 0
and thus g(α) is a root of f (x). Since f is irreducible, this means n = [F (g(α)) : F]. Note that
F (g(α)) ⊂ F (α). Therefore,

m = [F (α) : F] = [F (α) : F (g(α))][F (g(α)) : F] = [F (α) : F (g(α))] · n,

so n divides m, that is, deg f divides deg p.


Exercise 13.2.18.
(a) We follow the hint. Since k[t] is an UFD and k (t) is its field of fractions, then, by Gauss Lemma,
P(X ) − tQ(X ) is irreducible in k ((t))[X] is and only if it is irreducible in (k[t])[X]. Note that
(k[t])[X] = (k[X])[t]. Since P(X ) − tQ(X ) is linear in (k[X])[t], is clearly irreducible in (k[X])[t]
(i.e., in (k[t])[X]), hence in (k (t))[X]. Thus, P(X ) − tQ(X ) is irreducible in k (t). Now, x is clearly
P(x)
a root of P(X ) − tQ(X ) since P(x) − tQ(x) = P(x) − Q(x) = P(x) − P(x) = 0.
Q(x)
(b) Let n = max{degP(x), degQ(x)}. Write

P(x) = an x n + · · · + a1 x + a0 and Q(x) = bn x n + · · · + b1 x + b0,

where ai, bi ∈ k for all i, so at least one of an or bn is nonzero. The degree of P(X ) − tQ(X ) is clearly
≤ n, we prove is n. If an or bn is zero then clearly deg (P(X ) − tQ(X )) = n. Suppose an, bn , 0.
Then an, bn ∈ k, but t < k (as t ∈ k (x)), it cannot be that an = tbn . Thus (an − tbn )X n , 0, so the
degree of P(X ) − tQ(X ) is n.
(c) Since P(X ) − tQ(X ) is irreducible over k (t) and x is a root by part (a), then [k (x) : k (t)] =
degP(X ) − tQ(X ), and this degree equals max{degP(x), degQ(x)} by part (b).
Exercise 13.2.19.
(a) Fix α in K. Since K is (in particular) a commutative ring, we have α(a + b) = αa + αb and
α(λa) = λ(αa) for all a, b, λ ∈ K. If, in particular, λ ∈ F, we have the result.
 F. By part (a), for every α ∈ K we can associate a
(b) Fix a basis for K as a vector space over
F-linear transformation Tα . Denote by Tα the matrix of Tα with respect to the basis fixed above.
 
Then define ϕ : K → Mn (F) by ϕ(α) = Tα . We claim ϕ is an isomorphism. Indeed, if α, β ∈ K,
then T(α+β) (k) = (α + β)(k) = αk + βk = Tα (k) + Tβ (k) for every k ∈ K, hence T(α+β) = Tα + Tβ .
We also have T(αβ) (k) = (α β)(k) = α( βk) = TαTβ (k) for every k ∈ K, so T(αβ) = TαTβ . Thus
ϕ(α + β) = ϕ(α) + ϕ( β) and ϕ(α β) = ϕ(α)ϕ( β) (since the basis is fixed), so ϕ is an homomorphism.
Now, if ϕ(α) = ϕ( β), then αk = βk for every k ∈ K, so letting k = 1 we find that ϕ is injective.
Therefore, ϕ(K ) is isomorphic to a subfield of Mn (F), so the ring Mn (F) contains an isomorphic
copy of every extension of F of degree ≤ n.
Exercise 13.2.20.
The characteristic polynomial of A is p(x) = det(I x − A). For every k ∈ K, we have (Iα − A)k =
αk − Ak = αk − αk = 0, so det(Iα√3 − A) = 0 in K. √3 Therefore,
√3 p(α) = 0.
Now, consider
√3 the field Q(
√3 2) with basis
√3 {1, 2, 4} over
√3 Q. Denote the elements of this basis

by e1 = 1, e2 = 2 and e3 = 4. Let α = 2 and β = 1 + 2 + 4. Then α(e1 ) = e2 , α(e2 ) = e3 and
3

α(e3 ) = 2e1 . We also have β(e1 ) = e1 + e2 + e3 , β(e2 ) = 2e1 + e2 + e3 and β(e3 ) = 2e1 + 2e2 + e3 .
Thus, the associated matrices of the their linear transformations are, respectively,
0 0 2 1 2 2
Aα = ..1 0 0// Aβ = ..1 1 2// .
* + * +
and
,0 1 0- ,1 1 1-

9
13.2 Algebraic Extensions

The characteristic
√3 polynomial of Aα is x 3 − 2, hence is the monic polynomial of degree 3 satisfied by
α = 2. Furthermore, the characteristic polynomial of Aβ is x 3 − 3x 2 − 3x − 1, hence is the monic
√3 √3
polynomial of degree 3 satisfied by β = 1 + 2 + 4.
Exercise 13.2.21.
The matrix of the linear transformation "multiplication by α" on K is found by acting of α in the basis
√ √ √ √
!
a bD
1, D. We have α(1) = α = a + b D and α( D) = a D + bD. Hence the matrix is .
b a

!
a bD
Now let ϕ : K → M2 (Q) be defined by ϕ(a + b D) = .
b a
We have
√ √ a + c (b + d)D √ √
! ! !
a bD c dD
ϕ(a + b D + c + d D) = = + = ϕ(a + b D) + ϕ(c + d D),
b+d a+c b a d c

and
√ √ ac + bdD (ad + bc)D
! ! !
a bD c dD
ϕ((a + b D) · (c + d D)) = = ·
ad + bc ac + bdD b a d c
√ √
= ϕ(a + b D)ϕ(c + d D),

so ϕ is an homomorphism. Since K is a field, its ideals are {0} and K, so ker(ϕ) is trivial or K. Since
ϕ(K ) is clearly non-zero, then ker(ϕ) , K and thus ker(ϕ) = {0}. Hence, ϕ is injective. Therefore,
ϕ is an isomorphism of K with a subfield of M2 (Q).
Exercise 13.2.22.
Define ϕ : K1 × K2 → K1 K2 by ϕ(a, b) = ab. We prove that ϕ is F-bilinear. Let a, a1, a2 ∈ K and
b, b1, b2 ∈ K2 . Then

ϕ((a1, b) + (a2, b)) = ϕ(a1 + a2, b) = (a1 + a2 )b = a1 b + a2 b = ϕ(a1, b) + ϕ(a2, b),

and

ϕ((a,1 b) + (a, b2 )) = ϕ(a, b1 + b2 ) = a(b1 + b2 ) = ab1 + ab1 = ϕ(a, b1 ) + ϕ(a, b2 ).

We also have, for r ∈ F, ϕ(ar, b) = (ar)b = a(r b) = ϕ(r b). Therefore, ϕ is a F-bilinear map.
Hence, ϕ induces a F-algebra homomorphism Φ : K1 ⊗F K2 → K1 K2 . We use Φ to prove both
directions. Note that K1 ⊗F K2 have dimension [K1 : F][K2 : F] as a vector space over F.
First, we suppose [K1 K2 : F] = [K1 : F][K2 : F] and prove K1 ⊗F K2 is a field. In this case
K1 ⊗F K2 and K1 K2 have the same dimension over F. Let L = Φ(K1 ⊗F K2 ). We claim L = K1 K2 ,
i.e. Φ is surjective. Note that L contains K1 and K2 . Since L is a subring of K1 K2 containing K1 (or
K2 ), then L is a field (Exercise 16). Hence, L is a field containing both K1 and K2 . Since K1 K2 is
the smallest such field (by definition), we have L = K1 K2 . Therefore Φ is surjective, as claimed. So,
Φ is an F-algebra surjective homomorphism between F-algebras of the same dimension, hence is an
isomorphism. Thus, K1 ⊗F K2 is a field.
Now suppose that K1 ⊗F K2 is a field. In this case Φ is a field homomorphism. Therefore, Φ
is either injective or trivial. It is clearly nontrivial since Φ(1 ⊗ 1) = 1, so it is injective. Hence,
[K1 : F][K2 : F] ≤ [K1 K2 : F]. As we already have [K1 K2 : F] ≤ [K1 : F][K2 : F] (Proposition 21
of the book), the equality follows.

10
13.3 Classical Straightedge and Compass Constructions

13.3 Classical Straightedge and Compass Constructions


Exercise 13.3.1.
Suppose the 9-gon is constructible. It has angles of 40◦ . Since we can bisect an angle by straightedge
and compass, the angle of 20◦ would be constructible. But then cos 20◦ and sin 20◦ would be
constructible too, a contradiction (see proof of Theorem 24).
Exercise 13.3.2.
Let O, P, Q and R be the points marked in the figure below.

Then α = ∠QPO, β = ∠RQO, γ = ∠QRO, and θ is an exterior angle of 4PRO. Since 4PQO is
isosceles, then α = ∠QPO = ∠QOP. Since β is an exterior angle of 4PQO, it equals the sum of the
two remote interior angles, i.e., equals ∠QPO + ∠QOP. This two angles equals α, hence β = 2α.
Now, since 4QRO is isosceles, then β = γ. Finally, since θ is an exterior angle of 4PRO, equals the
sum of the two remote interior angles, which are α and γ. Therefore, θ = α + γ = α + β = 3α.
Exercise 13.3.3.
We follow the hint. The distances a, b, x, y and x − k are marked in the figure below.

From the figure, using similar triangles for (a), (b) and (c), and Pythagoras Theorem for (d), the
4 relations are clear. Hence, we have
√ √
1 − k2 b+k y 1 − k2
y= , x=a , = and (1 − k 2 ) + (b + k) 2 = (1 + a) 2 .
1+a 1+a x−k 3k
√ 3ky
So, 1 − k 2 = y(1 + a) = x−k implies 3k = (x − k)(1 + a). From the equation for x above, we find
a(b+a)
3k = ( 1+a − k)(1 + a) = a(b + k) − k (1 + a), so b + k = 4k+k a
a . Using this in the last equation
and reducing, we get

(1 − k 2 ) + (b + k) 2 = (1 + a) 2
4k + ka 2
⇒ (1 − k 2 ) + ( ) = (1 + a) 2
a
⇒ a2 (1 − k 2 ) + (4k + ka) 2 = a2 (1 + a) 2
⇒ a2 − (ka) 2 + (4k) 2 + 8k 2 a + (ka) 2 = a2 + 2a3 + a4
⇒ a4 + 2a3 − 8k 2 a − 16k 2 = 0.

11
13.4 Splitting Fields and Algebraic Closures

We let a = 2y to obtain
h4 + h3 − k 2 h − k 2 = 0.
We find h = k 2/3 , hence a = 2k 2/3 . From b = 4k+k
a
a
− k, we find b = 2k 1/3 . Therefore, we can
1/3 2/3
construct 2k and 2k using Conway’s construction.
Exercise 13.3.4.
Let p(x) = x 3 + x 2 − 2x − 1 and α = 2 cos(2π/7). By the Rational Root Theorem, if p has a root
in Q, it must be ±1 since it must divide its constant term. But p(1) = −1 and p(−1) = 1, so p is
irreducible over Q. Therefore, α is of degree 3 over Q, hence [Q(α) : Q] cannot be a power of 2.
Since we can’t construct α, the regular 7-gon is not constructible by straightedge and compass.
Exercise 13.3.5.
Let p(x) = x 2 + x − 1 = 0 and α = 2 cos(2π/5). By the Rational Root Theorem, if p has a root in Q, it
must be ±1. Since p(1) = 1 and p(−1) = −1, p is irreducible over Q. Hence, α is of degree 2 over Q,
angle by straightedge and compass, so β = cos(2π/5) is also
so it is constructible. We can bisect an p
constructible. Finally, as sin(2π/5) = 1 − cos2 (2π/5), sin(2π/5) is also constructible. Therefore,
the regular 5-gon is constructible by straightedge and compass.

13.4 Splitting Fields and Algebraic Closures


Exercise 13.4.1.
√4 √4 √4 √4
f (x) = x 4 − 2. The roots of f are 2, − 2, i √2 and −i 2. Hence,
Let √ √4 the√splitting√field of f is
Q(i, 2). √So, the splitting field of f has degree [Q(i, 2) : Q] = [Q(i, 2)√: Q( 2)][Q( 2) : Q] over
4 4 4 4

= 4. Furthermore,
4 4
Q. Since 2√is a root of the irreducible polynomial x√4 −2 over Q, then [Q( 2) : Q] √ √4
since i < Q( 2),√then x 2 + 1 is irreducible over Q( 2) having i as a root, so [Q(i, 2) : Q( 2)] = 2.
4 4 4

Therefore, [Q(i, 2) : Q] = 8.
4

Exercise 13.4.2.
Let f (x) = x 4 + 2. Let K be the splitting field of f and let L be the splitting field of x 4 − 2,

that√is,
√4
L = Q(i, 2) (last exercise). We claim K = L, so that [K : Q] = 8 by last exercise. Let ζ = 2 + i 22 .
2

First we prove ζ ∈ L and ζ ∈ K, then we prove √4 K = L. √


√We prove ζ ∈ L. This is easy. Let θ = 2. Since θ ∈ L, then θ 2 = 2 ∈ L. We also have i ∈ L,
so 2, i ∈ L implies ζ ∈ L. √
We prove ζ ∈ K. We have to prove i ∈ K and 2 ∈ K. Let α be a root of x 4 + 2, so that α 4 = −2.
Let β be a root of x 4 − 1, so that β 4 = 1. Then (α β) 4 = α 4 β 4 = −2, hence α β is also a root of
x 4 + 2. Since the roots of x 4 − 1 are ±1, ±i, the roots of x 4 + 2 are ±α and ±iα. Since K is generated
over Q by there roots, then iα/α = i ∈ K. Now √ let γ = √ α ∈ K. Since γ = α = −2, then γ is a
2 2 4

root of x + 2. Since the√roots of x + 2 are i 2 and −i 2, then γ is one of this roots. In either case
2 2

γ/i ∈ K, which implies 2 ∈ K. Therefore, ζ ∈ K.


Now we prove L = K proving both inclusions.
Let α be a root of x 4 + 2 and θ be a root of x 4 − 2. Then α 4 = −2 and θ 4 = 2. Note that ζ 2 = i, so
ζ = −1. Hence, (ζ θ) 4 = ζ 4 θ 4 = −2, so ζ θ is a root of x 4 + 2. Then, as we proved earlier, the roots
4

of x 4 + 2 are ±ζ θ and ±iζ θ. We also have (ζ α) 4 = ζ 4 α 4 = 2, so ζ α is a root of x 4 − 2. Then, by


last exercise, the roots of x 4 − 2 are ±ζ α and ±iζ α. Now, since ζ and α are in K, we have ζ α ∈ K.
We also have i ∈ K, so all roots of x 4 − 2 are in K. Since L is generated by these roots, then L ⊂ K.
Similarly, ζ and θ are in L, so ζ θ ∈ L; since i ∈ L, then all roots of x 4 + 2 are in L. Since K is
generated by these roots, we have K ⊂ L. Therefore, K = L, so [K : Q] = 8.

12
13.5 Separable and Inseparable Extension

Exercise 13.4.3. √
Let f (x) = x 4 + x 2 + 1. Note that f (x) = (x 2 + x + 1)(x 2 − x + 1), so the roots of f are ± 21 ± i 23 .

Let w = 1
2√− i 3
2 , so this roots are w, −w, w, −w, where w denotes the complex conjugate of w (i.e.,
w = 12 + i 23 ). Hence, the splitting field of f is Q(w, w). Since w + w = 1, then Q(w, w) = Q(w).
Furthermore, w is a root of x 2 − x + 1, that is irreducible over Q since w < Q. Therefore, the degree
of the splitting field of f is [Q(w) : Q] = 2.
Exercise 13.4.4.
√3 √3 √3
Let f (x) = x 6 − 4. Note that f (x) = (x 3 − 2)(x 3 + 2). The roots of x 3 − 2 are 2, ζ 2 and ζ 2 √2,
where ζ is the primitive 3’rd root of unity, i.e., ζ = exp(2πi/3) = cos(2π/3) + i sin(2π/3) = − 21 + 23 .
√3 √3 √3
Furthermore,
√3 the roots √of x 3 + 2 are − 2,√ −ζ 2 and√ −ζ 2 √2. Therefore, the splitting
√3 field of f is
Q(ζ, 2). Then [Q(ζ, 2) : Q] = [Q(ζ, 2) √
3 3 3 3
: Q( 2)][Q( 2) : Q]. We have that 2 is a root of
3 over Q. Furthermore, ζ is a root of
3
the irreducible polynomial x 3 √ − 2 over Q, so 2 has degree √
x 2 + x + 1, irreducible ζ
3 3
√3 over Q( 2), so has degree 2 over Q( 2). Hence, the degree of the splitting
field of f is [Q(ζ, 2) : Q] = 6.
Exercise 13.4.5.
We follow the hint. First suppose that K is a splitting field over F. Hence, there exists f (x) ∈ F[x]
such that K is the splitting field of f . Let g(x) be an irreducible polynomial in F[x] with a root α ∈ K.
Let β be any root of g. We prove β ∈ K, so that g splits completely in K[x]. By Theorem 8, there is

an isomorphism ϕ : F (α) − → F ( β) such that ϕ(α) = β. Furthermore, K (α) is the splitting field for
f over F (a), and K ( β) is the splitting field for f over F ( β). Therefore, by Theorem 28, ϕ extends

to an isomorphism σ : K (α) − → K ( β). Since K = K (α), then [K : F] = [K (α) : F] = [K ( β) : F],
so K = K ( β). Thus, β ∈ K.
Now suppose that every irreducible polynomial in F[x] that has a root in K splits completely in
K[x]. Since [K : F] is finite, then K = F (α1, . . . , α n ) for some α1, . . . , α n . For every 1 ≤ i ≤ n, let
pi be the minimal polynomial of αi over F, and let f = p1 p2 · · · pn . Since every αi is in K, every pi
has a root in K, hence splits completely in K. Therefore, f splits completely in K and K is generated
over F by its roots, so K is the splitting field of f (x) ∈ F[x].
Exercise 13.4.6.
(a) Let K1 be the splitting field of f 1 (x) ∈ F[x] over F and K2 the splitting field of f 2 (x) ∈ F[x]
over F. Thus, K1 is generated over F by the roots of f 1 , and K2 is generated over F by the roots of
f 2 . Therefore, f 1 f 2 splits completely in K1 K2 and K1 K2 is generated over F by its roots, hence is the
splitting field of f 1 f 2 (x) ∈ F[x].
(b) We follow the hint. By last exercise, we have to prove that every irreducible polynomial in F[x]
that has a root in K1 ∩ K2 splits completely in (K1 ∩ K2 )[x]. So, let f (x) be an irreducible polynomial
in F[x] that has a root, say α, in K1 ∩ K2 . By last exercise, f splits completely in K1 and splits
completely in K2 . Since K1 and K2 are contained in K, by the uniqueness of the factorization of f in
K, the roots of f in K1 must coincide with its roots in K2 . Hence, f splits completely in (K1 ∩ K2 )[x].

13.5 Separable and Inseparable Extension


Exercise 13.5.1.
Let f (x) = an x n + · · · + a1 x + a0 and g(x) = bm x m + · · · + b1 x + b0 be two polynomials. Suppose,
without any loss of generality, that n ≥ m. Thus, we can write g(x) = bn x n +· · ·+b1 x+b0 , where some
of the last coefficients bi could be zero. We have f (x)+g(x) = (an +bn )x n +· · ·+(a1 +b1 )x+(a0 +b0 ),
so

D x ( f (x) + g(x)) = n(an + bn )x n−1 + · · · + 2(a2 + b2 )x + (a1 + b1 ) = D x ( f (x)) + D x (g(x)).

13
13.5 Separable and Inseparable Extension

Now we prove the formula for the product. Let cn =


Pn
k=0 ak bn−k , so that

n
X n
X 2n X
X l 2n
X
f (x)g(x) = ( ak x k )( bk x k ) = ( ak bl−k )x l = cl x l .
k=0 k=0 l=0 k=0 l=0

Hence,
2n
X 2n−1
X
D x ( f (x)g(x)) = D x ( cl x l ) = (l + 1)cl+1 x l,
l=0 l=0

so the coefficient of in D x ( f (x)g(x)) is (l + 1)cl+1 .


xl
Now, we have D x ( f (x)) = nan x n−1 + · · · + 2a2 x + a1, and D x (g(x)) = nbn x n−1 + · · · + 2b2 x + b1 .
So (recall product of polynomials, page 295 of the book)
n
X n
X
D x ( f (x))g(x) = ( kak x k−1 )( bk x k )
k=1 k=0
n−1
X n
X 2n−1
X l
X
=( (k + 1)ak+1 x k )( bk x k ) = ( (k + 1)ak+1 bl−k )x l,
k=0 k=0 l=0 k=0

and
n
X n
X
f (x)D x (g(x)) = ( ak x k )( kbk x k−1 )
k=0 k=1
Xn n−1
X 2n−1
X l
X
=( k
ak x )( (k + 1)bk+1 x ) = k
( ak (l − k + 1)bl−k+1 )x l .
k=0 k=0 l=0 k=0

Therefore, the coefficient of x l in D x ( f (x))g(x) + D x (g(x)) f (x) is


l
X l
X
( (k + 1)ak+1 bl−k ) + ( (l − k + 1)ak bl−k+1 )
k=0 k=0
l−1
X l
X
= (l + 1)al+1 b0 + ( (k + 1)ak+1 bl−k ) + ( (l − k + 1)ak bl−k+1 ) + (l + 1)a0 bl+1
k=0 k=1
Xl l
X
= (l + 1)al+1 b0 + ( kak bl−k+1 ) + ( (l − k + 1)ak bl−k+1 ) + (l + 1)a0 bl+1
k=1 k=1
Xl
= (l + 1)al+1 b0 + ( (l + 1)ak bl−k+1 ) + (l + 1)a0 bl+1
k=1
l+1
X
= (l + 1)( ak bl−k+1 ) = (l + 1)cl+1 .
k=0

Since all their coefficients are equal, we conclude D x ( f (x)g(x)) = D x ( f (x))g(x) + D x (g(x)) f (x).
Exercise 13.5.2.
The polynomials x and x + 1 are the only (non-constant, i.e. , 0, 1) polynomials of degree 1 over
F2 ; they are clearly irreducible. A polynomial f (x) ∈ F2 [x] of degree 2 is irreducible over F2 if
and only if it does not have a root in F2 , that is, exactly when f (0) = f (1) = 1. Hence, the only
irreducible polynomial of degree 2 over F2 is x 2 + x + 1. Now, for a polynomial f (x) ∈ F2 [x] of

14
13.5 Separable and Inseparable Extension

degree 4 to be irreducible, it must have no linear or quadratic factors. We can also apply the condition
f (1) = f (0) = 1 to discard the ones with linear factors. Furthermore, f must have an odd number of
terms (or it will be 0), and must have constant term 1 (or x will be a factor). We are left with

x4 + x3 + x2 + x + 1 x4 + x3 + 1
x4 + x2 + 1 x 4 + x + 1.

For any of this polynomials to be irreducible, it can’t be factorized as two quadratic irreducible factors.
Since x 2 + x +1 is the only irreducible polynomial of degree 2 over F2 , only (x 2 + x +1) 2 = x 4 + x 2 +1 of
this four is not irreducible. Hence, the irreducible polynomials of degree 4 over F2 are x 4 +x 3 +x 2 +x+1,
x 4 + x 3 + 1 and x 4 + x + 1.
Now, since x + 1 = x − 1 in F2 , we have (x + 1)(x 4 + x 3 + x 2 + x + 1) = x 5 − 1. We also calculate
(x 2 + x + 1)(x 4 + x + 1)(x 4 + x 3 + 1) = x 10 + x 5 + 1. So, the product of all this irreducible polynomials
is
x(x + 1)(x 2 + x + 1)(x 4 + x + 1)(x 4 + x 3 + 1)(x 4 + x 3 + x 2 + x + 1)
= x(x 5 − 1)(x 10 + x 5 + 1) = x 16 − x.

Exercise 13.5.3.
We follow the hint. Suppose d divides n, so that n = qd for some q ∈ Z. Then x n − 1 = x qd − 1 =
(x d − 1)(x qd−d + x qd−2d + . . . + x d + 1). So x d − 1 divides x n − 1.
Conversely, suppose d does not divide n. Then n = qd + r for some q, r ∈ Z with 0 < r < d.
Thus x n − 1 = (x qd+r − x r ) + (x r − 1) = x r (x qd − 1) + (x r − 1) = x r (x d − 1)(x qd−d + x qd−2d +
. . . + x d + 1) + (x r − 1). Since x d − 1 divides the first term, but doesn’t divide x r − 1 (as r < d), then
x d − 1 does not divide x n − 1.
Exercise 13.5.4.
The first assertion is analogous to the last exercise.
d
Now, F p d is defined as the field whose pd elements are the roots of x p − x over F p . Similarly
is defined F p n . Take a = p. So, d divides n if and only if pd − 1 divides pn − 1, and that occurs
d n
exactly when x p −1 − 1 divides x p −1 − 1 (by last exercise). Thus, if d divides n, any root of
d d n n
x p − x = x(x p −1 − 1) must be a root of x p − x = x(x p −1 − 1), hence F p d ⊆ F p n . Conversely, if
d n
F p d ⊆ F p n , then x p −1 − 1 divides x p −1 − 1, so d divides n.
Exercise 13.5.5.
Let f (x) = x p − x + a. Let α be a root of f (x). First we prove f is separable. Since (α + 1) p − (α +
1) + a = α p + 1 − α − 1 + a = 0, then α + 1 is also a root of f (x). This gives p distinct roots of f (x)
given by α + k with k ∈ F p , so f is separable.
Now we prove f is irreducible. Let f = f 1 f 2 · · · f n where f i (x) ∈ F p [x] is irreducible for all
1 ≤ i ≤ n. Let 1 ≤ i < j ≤ n and let αi be a root of f i and α j be a root of f j , so that f i is the minimal
polynomial of αi and f j the minimal polynomial of α j . We prove deg f i = deg f j . Since αi is a root
of f i , it is a root of f , hence there exists k 1 ∈ F p such that αi = α + k 1 . Similarly, there exists k 2 ∈ F p
such that α j = α + k 2 . Thus, αi = α j + k1 − k 2 , so f i (x + k 1 − k 2 ) is irreducible having α j as a root,
so it must be its minimal polynomial. Hence, f i (x + k 1 − k 2 ) = f j (x), so deg f i = deg f j , as claimed.
Since i and j were arbitrary, then all f i are of the same degree, say q. Then p = deg f = nq, so n = 1
or n = p (as p is prime). If n = p, then all roots of f are in F p , so α ∈ F p and thus 0 = α p − α + a = a,
contrary to the hypothesis. Therefore, n = 1, so f is irreducible.

15
13.5 Separable and Inseparable Extension

Exercise 13.5.6.
n n
By definition, F p n is the field whose pn elements are the roots of x p − x over F p . Since x p − 1 =
n
x(x p −1 − 1), clearly Y
n
x p −1 − 1 = (x − α).
α∈F×p n

Set x = 0. Then Y n −1
Y
−1 = (−α) = (−1) p α
α∈F×p n α∈F×p n
n −1 n −1 n −1
Y
⇒ (−1) p (−1) = (−1) p (−1) p α
α∈F×p n
n
Y
⇒ (−1) p = α.
α∈F×p n

Hence, the product of the nonzero elements is +1 if p = 2 and −1 is p is odd. For p odd and n = 1,
we have Y
−1 = α,
α∈F×p

so taking module p we find [1][2] · · · [p − 1] = [−1], i.e., (p − 1)! ≡ −1 (mod p).


Exercise 13.5.7.
Let a ∈ K such that a , bp for every b ∈ K. Let f (x) = x p − a. We prove that f is irreducible
and inseparable. If α is a root of x p − a, then x p − a = (x − α) p , so α is a multiple root of f (with
multiplicity p), hence f is inseparable. Now, let g(x) be an irreducible factor of f (x). Note that
α < K, otherwise a = α p , contrary to the hypothesis. Then g(x) = (x − α) k for some k ≤ p. Using
the binomial theorem, we have

g(x) = (x − α) k = x k − kαx k−1 + · · · + (−α) k .

Therefore, kα ∈ K. Since α < K, then k = p, so g = f . Hence, f is irreducible. We conclude that


K (α) is an inseparable finite extension of K.
Exercise 13.5.8.
Let f (x) = an x n + . . . + a1 x + a0 ∈ F p [x]. Since F p has characteristic p, then (a + b) p = a p + bp for
any a, b ∈ F p . Easily we can generalize this to a finite number of terms, so that (x 1 + · · · + x n ) p =
p p
x 1 + · · · + x n for any x 1, · · · , x n ∈ F p . Furthermore, by Fermat’s Little Theorem, a p = a for every
a ∈ F p . So, over F p , we have
p p p
f (x) p = (an x n + . . . + a1 x + a0 ) p = an x np + . . . + a1 x p + a0 = an x np + . . . + a1 x p + a0 = f (x p ).

Exercise 13.5.9.
By the binomial theorem, we have
pn !
X pn i
(1 + x) pn
= x,
i=0
i
 
so the coefficient of x pi in the expansion of (1 + x) pn is pn pi .
Since F p has characteristic p, we have (1 + x) pn = 1 + x pn = (1 + x p ) n , so over F p this is the
coefficient of (x p ) i in (1 + x p ) n . Furthermore, (1 + x) pn = (1 + x p ) n implies

16
13.6 Cyclotomic Polynomials and Extensions

n ! pn !
X n p i X pn i
(1 + x p ) n = (x ) = x = (1 + x) pn
i=0
i i=0
k
 pn n
over F p , hence pi ≡ i (mod p).
Exercise 13.5.10.
p p p
This is equivalent to prove that for any prime number p, we have f (x 1, x 2, . . . , x n ) p = f (x 1 , x 2 , . . . , x n )
in F p [x 1, x 2, . . . , x n ]. Let
γ γ
X
f (x 1, x 2, . . . , x n ) = aγ1,...,γn x 11 . . . x nn
γ1,...,γn =0

be an element of F p [x 1, x 2, . . . , x n ].
p p
Since F p has characteristic p, then (x 1 + · · · + x n ) p = x 1 + · · · + x n for any x 1, · · · , x n ∈ F p .
Furthermore, by Fermat’s Little Theorem, a p = a for every a ∈ F p . Hence, over F p we have
γ γ γ γ
X X
f (x 1, x 2, . . . , x n ) p = ( aγ1,...,γn x 11 . . . x nn ) p = (aγ1,...,γn x 11 . . . x nn ) p
γ γ
X X
p pγ pγ p p p
= aγ1,...,γn (x 11 . . . x nn ) p = aγ1,...,γn (x 1 1 . . . x n n ) = f (x 1 , x 2 , . . . , x n ).

Exercise 13.5.11.
Let f (x) ∈ F[x] with no repeated irreducible factors in F[x]. We can suppose f is monic. Then
f = f 1 f 2 · · · f n for some monic irreducible polynomials f i (x) ∈ F[x]. Since F is perfect, f is
separable, hence all f i has distinct roots. Thus, f splits in linear factors in the closure of F, hence
splits in linear factors in the closure of K. Therefore, f (x) has no repeated irreducible factors in
K[x].

13.6 Cyclotomic Polynomials and Extensions


Exercise 13.6.1.
Since (ζ m ζ n ) mn = 1, then ζ m ζ n is an mnth root of unity. Now, let 1 ≤ k < mn. Then (ζ m ζ n ) k = ζ m
k ζk.
n
For a contradiction, suppose ζ m ζ n = 1. Then, ζ m = 1 and ζ n = 1, hence m divides k and n divides
k k k k

k, so mn divides k (as m , n). Since 1 ≤ k < mn, this is impossible. So, (ζ m ζ n ) k , 1 for all
1 ≤ k < mn and thus the order of ζ m ζ n is mn. Therefore, ζ m ζ n generates the cyclic group of all mnth
roots of unity, that is, ζ m ζ n is a primitive mnth root of unity.
Exercise 13.6.2.
Since (ζ nd ) (n/d) = ζ nn = 1, then ζ nd is an (n/d) th root of unity. Now let 1 ≤ k < (n/d). Then
(ζ nd ) k = ζ nkd . Since 1 ≤ kd < n, then ζ nkd , 1, so (ζ nd ) k , 1. Hence, the order of ζ nd is (n/d), so it
generates the cyclic group of all (n/d) th roots of unity, that is, ζ nd is a primitive (n/d) th root of unity.
Exercise 13.6.3.
Let F be a field that contains the nth roots of unity for n odd and let ζ be a 2nth root of unity. If
ζ n = 1, then ζ ∈ F, so suppose ζ n , 1. Since ζ 2n = 1, then ζ n is a root of x 2 − 1. Since the roots of
this polynomial are 1 and −1, and ζ n , 1, then ζ n = −1. Hence, (−ζ ) n = (−1) n (ζ ) n = (−1) n+1 = 1
(since n is odd), so −ζ ∈ F. Since F is a field, then ζ ∈ F.
Exercise 13.6.4.
k
Let F be a field with char F = p. The roots of unity over F are the roots of x n − 1 = x p m − 1 =
k
(x m − 1) p , so are the roots of x m − 1. Now, since m is relatively prime to p, so is x m − 1 and

17
13.6 Cyclotomic Polynomials and Extensions

its derivative mx m−1 , so x m − 1 has no multiple roots. Hence, the m different roots of x m − 1 are
precisely the m distinct nth roots of unity over F.
Exercise 13.6.5.

We use the inequality ϕ(n) ≥ n/2 for all n ≥ 1. Let K be an extension of Q with infinitely many
roots of unity. Let N ∈ N. Then, there exits n ∈ N such that n > 4N 2 and there exits some nth root of

unity ζ ∈ K. Then [K : Q] ≥ [Q(ζ ) : Q] = ϕ(n) ≥ n/2 > N. Since N was arbitrary, we conclude
that [K : Q] > N for all N ∈ N, so [K : Q] is infinite. Therefore, in any finite extension of Q there
are only a finite number of roots of unity.
Exercise 13.6.6.
Since Φ2n (x) and Φn (−x) are irreducible, they are the minimal polynomial of any of its roots. Thus,
it is sufficient to find a common root for both. Let ζ2 be the primitive 2th root of unity and let ζ n be a
primitive nth root of unity. Note that ζ2 = −1, so that ζ2 ζ n = −ζ n . Since n is odd, 2 and n are relatively
prime. So, by Exercise 1, ζ2 ζ n is a primitive 2nth root of unity, i.e, a root of Φ2n (x). Furthermore,
−ζ n is clearly a root of Φn (−x). Thus, −ζ n is a common root for both Φ2n (x) and Φn (−x), hence
Φ2n (x) = Φn (−x).
Exercise 13.6.7.
The Möbius Inversion Formula sates that if f (n) is defined for all nonnegative integers and F (n) =
d |n f (d), then f (n) = d |n µ(d)F ( d ). So lets start with the formula
P P n

Y
xn − 1 = Φd (x).
d |n

We take natural logarithm in both sides and obtain


Y X
ln(x n − 1) = ln( Φd (x)) = ln Φd (x).
d |n d |n

So, we use the Möbius Inversion Formula for f (n) = ln Φn (x) and F (n) = ln(x n − 1) to obtain

ln(x n/d − 1) µ(d) .


X X
ln Φn (x) = µ(d) ln(x n/d − 1) =
d |n d |n

Hence, taking exponentials we obtain

ln(x n/d − 1) µ(d) ) = (x n/d − 1) µ(d) = (x d − 1) µ(n/d) .


X Y Y
Φn (x) = exp(
d |n d |n d |n

Exercise 13.6.8.
x `−1 (x−1) `
(a) Since p is prime, in F p [x] we have (x − 1) p = x p − 1, so Φ` (x) = x−1 = x−1 = (x − 1) l−1 .
(b) Note that ζ has order ` as being a primitive ` th root of unity. Since 1 mod `, then p f − 1 = q`
pf ≡
f
for some integer q, hence ζ p −1 = ζ q` = 1, so ζ ∈ F p f . Now we prove that f is the smallest integer
n
with that property. Suppose ζ ∈ F p n for some n. Then ζ is a root of x p −1 − 1, hence ` divides pn − 1
(see Exercise 13.5.3). Since f is the smallest power of p such that p f ≡ 1 mod `, is the smallest
integer such that ` divides p f − 1, so n ≥ l, as desired. This in fact proves that F p (ζ ) = F p f , so the
minimal polynomial of ζ over F p has degree f .
(c) Since ζ a ∈ F p (ζ ), clearly F p (ζ a ) ⊂ F p (ζ ). For the other direction we follow the hint. Let b the
the multiplicative inverse of a mod `, i.e ab ≡ 1 mod `. Then (ζ a ) b = ζ, so ζ ∈ F p (ζ a ) and thus
F p (ζ ) ⊂ F p (ζ a ). The equality follows.

18
13.6 Cyclotomic Polynomials and Extensions

Now, consider Φ` (x) as a polynomial over F p [x]. Let ζi for 1 ≤ i ≤ ` be ` distinct primitive ` th
roots of unity. The minimal polynomial of each ζi has degree f by part (b). Hence, the irreducible
factors of Φ` (x) have degree f . Since Φ` have degree ` − 1, then there must be `−1 f factors, and all
of them are different since Φ` (x) is separable.
(d) If p = 7, then Φ7 (x) = (x − 1) 6 by part (a). If p ≡ 1 mod 7, then f = 1 in (b) and all roots have
degree 1, so Φ7 (x) splits in distinct linear factors. If p ≡ 6 mod 7, then f = 2 is the smallest integer
such that p f = p2 ≡ 36 ≡ 1 mod 7, so we have 3 irreducible quadratics. If p ≡ 2, 4 mod 7, then
f = 3 is the smallest integer such that p3 ≡ 23, 43 ≡ 8, 64 ≡ 1 mod 7, so we have 2 irreducible cubics.
Finally, if p ≡ 3, 5 mod 7, then f = 6 is the smallest integer such that p6 ≡ 36, 56 ≡ 729, 15626 ≡ 1
mod 7, hence we have an irreducible factor of degree 6.
Exercise 13.6.9.
Let A be an n by n matrix over C for which Ak = I for some integer k ≥ 1. Then the minimal
polynomial of A divides x k − 1. Since we are working over C, there are k distinct roots of this
polynomial, so the minimal polynomial of A can be split in linear factors. Hence, A is diagonalizable.
1 α
!
Now consider A = where α is an element of a field of characteristic p.
0 1
!
1 nα
Computing powers of A, we can prove (by induction) that A = n for every positive integer
0 1
n. Since pα = 0, then Ap = I. Now, if A is diagonalizable, there exists some non-singular matrix P
such that A = PDP−1 , where D is a diagonal matrix whose diagonal entries are the eigenvalues of A.
Since A has only one eigenvalue 1, then D = I, and thus A = PI P−1 = I. So, if A is diagonalizable,
it must be α = 0.
Exercise 13.6.10.
Let a, b ∈ F p n . Then ϕ(a + b) = (a + b) p = a p + bp = ϕ(a) + ϕ(b), and ϕ(ab) = (ab) p = a p bp =
ϕ(a)ϕ(b), so ϕ is and homomorphism. Moreover, if ϕ(a) = 0, then a p = 0 implies a = 0. Hence, ϕ
is injective. Since F p n is finite, then ϕ is also surjective so it is an isomorphism. Furthermore, since
n n
every element of F p n is a root of x p − x, then ϕn (a) = a p = a for all a ∈ F p n , so ϕn is the identity
m
map. Now, let m be an integer such that ϕm is the identity map. Then a p = a for all a ∈ F p n , so
m
every element of F p n must be a root of x p − x. Hence, F p n ⊂ F p m and thus n divides m (Exercise
13.5.4), so n ≤ m.
Exercise 13.6.11.
Note that the minimal polynomial of ϕ is x n −1, for if ϕ satisfies some polynomial x n−1 +· · ·+ a1 x + a0
n−1
of degree n − 1 (or less) with coefficients in F p , then x p + · · · + a1 x p + a0 for all x ∈ F p n , which
is impossible. Since F p n has degree n as a vector space over F p , then x n − 1 is also the characteristic
polynomial of ϕ, hence is the only invariant factor. Therefore, the rational canonical form of ϕ over
F p is the companion matrix of x n − 1, which is

0 0 ··· 0 1
*. +
..1 0 · · · 0 0//
..0 1 · · · 0 0/// .
.. .. .... . ./
.. . . .. .. //
,0 0 · · · 1 0-

Exercise 13.6.12.
We’ll work over the algebraic closure of F p n , to ensure the field to contain all eigenvalues. In last
exercise we proved that the minimal and characteristic polynomial of ϕ is x n − 1. Moreover, the
eigenvalues of ϕ are the nth roots of unity. We use Exercise 4 and write n = pk m for some prime p

19
13.6 Cyclotomic Polynomials and Extensions

k
and some m relatively prime to p, so that x n − 1 = (x m − 1) p and we get exactly m distinct nth roots
of unity, each one with multiplicity pk . Since all the eigenvalues are zeros of both the minimal and
characteristic polynomial of multiplicity pk , we get m Jordan blocks of size pk . Now, fix a primitive
mth root of unity, say ζ. Then, each Jordan block each of the form

ζi 1 0 ··· 0 0
.. 0 ζi 1 ··· 0 0 //
*. +
.0 0 ζi ··· 0 0 //
Ji = .. . .. .. .. .. .. //
.. .. . . . . . //
.. 0 0 0 · · · ζi 1 //
,0 0 0 ··· 0 ζ i-

for some 0 ≤ i ≤ m − 1. We already know the Jordan canonical form is given by

J 0 ··· 0
*. 0 +/
.. 0 J1 · · · 0 //
.. 0 0 ··· 0 // .
.. .. .. . . ..
.. . . .
//
/
,0 0 ··· Jm−1 -

Exercise 13.6.13.
(a) Z is a division subring of D, and it is commutative by definition of the center, so Z is a field.
Since it is finite, its prime subfield is F p for some prime p, so it is isomorphic to F p m for some integer
m. Let q = pn , so that Z is isomorphic to Fq . Since D is a vector space over Z, then |D| = q n for
some integer n.
(b) Let x ∈ D× and let CD (x) be the set of the elements in D that commutes with x. Clearly
Z ⊂ CD (x). We prove that every element a ∈ CD (x) has an inverse in CD (x). Since a ∈ CD (x),
then ax = xa. Since D is a division ring, then a−1 ∈ D. Moreover, we have a−1 ax = a−1 xa and
thus x = a−1 xa, so xa−1 = a−1 x and a−1 ∈ CD (x). Hence, CD (x) is a division ring. Now, since
Z ⊂ CD (x), then CD (x) is a Z-vector space, so |CD (x)| = q m for some integer m. If x < Z, then
CD (x) is a proper subset of D and hence m < n.
(c) The class equation for the group D× is
r
X
|D× | = | Z (D × )| + |D× : CD × (x i )|,
i=1

where the x i are the representatives of the distinct conjugacy class. By (a) we have |D× | = q n − 1,
qn − 1 qn − 1
| Z (D × )| = q − 1 and |CD × (x i )| = q mi − 1. Then |D× : CD × (x i )| = = m . Replacing
|CD × (x i )| q i − 1
in the class equation we obtain
r r
X qn − 1 X qn − 1
q − 1 = (q − 1) +
n
= (q − 1) + .
i=1
|CD × (x i )| i=1
q mi − 1

qn − 1
(d) Since |D × : CD × (x i )| is an integer, then |D× : CD × (x i )| = is an integer. Hence, q mi − 1
q mi − 1
divides q n − 1, so (Exercise 13.5.4) mi divides n. Since mi < n (no x i is in Z), no mith root of unity
is a nth root of unity. Therefore, as Φn (x) divides x n − 1, it must divide (x n − 1)/(x mi − 1) for
i = 1, 2, . . . , r.. Letting x = q we have Φn (q) divides (q n − 1)/(q mi − 1) for i = 1, 2, . . . , r.

20
13.6 Cyclotomic Polynomials and Extensions

(e) From (d), Φn (q) divides (q n − 1)/(q mi − 1) for i = 1, 2, . . . , r, so the class equation in (c) implies
Φn (q) divides q − 1. Now, let ζ , 1 be a nth root of unity. In the complex plane q is closer to 1 that
ζ is, so |q − ζ | > |q − 1| = q − 1. Since Φn (q) = ζ primitive (q − ζ ) divides q − 1, this is impossible
Q
unless n = 1. Hence, D = Z and D is a field.
Exercise 13.6.14.
We follow the hint. Let P(x) = x n + · · · + a1 x + a0 be a monic polynomial of degree ≥ 1 over Z. For a
contradiction, suppose there are only finitely many primes dividing the values P(n), n = 1, 2, . . ., say
p1, p2, . . . , pk . Let N be an integer such that P(N ) = a , 0. Let Q(x) = a−1 P(N + ap1 p2 . . . pk x).
Then, using the binomial theorem, we have

Q(x) = a−1 P(N + ap1 p2 . . . pk x)


= a−1 ((N + ap1 p2 . . . pk x) n + · · · + a1 (N + ap1 p2 . . . pk x) + a0 )
= a−1 (N n + an−1 N n−1 + · · · + a1 N + a0 + R(x))
= a−1 (P(N ) + R(x))
= 1 + a−1 R(x)

for some polynomial R(x) ∈ Z[x] divisible by ap1 p2 . . . pk . Hence, Q(x) ∈ Z[x]. Moreover, for
all n ∈ Z+ , P(N + ap1 p2 . . . pk n) ≡ a (mod p1, p2, . . . , pk ), so Q(n) = a−1 P(N + ap1 p2 . . . pk n) ≡
a−1 a = 1 (mod p1, p2, . . . , pk ). Now let m be a positive integer such that |Q(m)| > 1, so that
Q(m) ≡ 1 (mod pi ) for all i. Therefore, none of the pi ’s divide Q(m). Since |Q(m)| > 1, there exists
a prime q , pi for all i such that q divides Q(m). Then q divides aQ(m) = P(N + ap1 p2 . . . pk m),
contradicting the fact that only the primes p1, p2, . . . , pk divides the numbers P(1), P(2), . . ..
Exercise 13.6.15.
We follow the hint. Since Φm (a) ≡ 0 (mod p), then a m ≡ 1 (mod p). Hence, there exist b such that
ba ≡ 1 mod p (indeed, b = a m−1 ), so a is relatively prime to p. We prove that the order of a is m.
For a contradiction, suppose a d ≡ 1 (mod p) for some d dividing m, so that Φd (a) ≡ 0 (mod p) for
some d < m. Thus, a is a multiple root of x m − 1, so is also a root of its derivative ma m−1 . Hence,
ma m−1 ≡ 0 mod p, impossible since p does not divide m nor a. Therefore, the order of a in (Z/pZ) ×
is precisely m
Exercise 13.6.16.
Let p be an odd prime dividing Φm (a). If p does not divide m, then, by (c), a is relatively prime to p
and the order of a in F×p is m. Since |F×p | = p − 1, this implies m divides p − 1, that is, p ≡ 1 (mod m).
Exercise 13.6.17.
By Exercise 14, there are infinitely many primes dividing Φm (1), Φm (2), Φm (3), . . .. Since only
finitely of them can divide m, then, by Exercise 16, there must exists infinitely many primes p with
p ≡ 1 (mod m).

Please send comments, suggestions and corrections by e-mail, or at website.


https://positron0802.wordpress.com
positron0802@mail.com

21

Potrebbero piacerti anche